If $bigcup_{i=1}^infty A_i$ has cardinality $kappa$, then some $A_i$ has cardinality $kappa$?












2












$begingroup$


Is the following claim true?




If $bigcup_{i=1}^infty A_i$ has cardinality $kappa$, then some $A_i$ has cardinality $kappa$. Here $kappa$ is uncountable.




I'm interested in the particular case $kappa=c:=|mathbb{R}|$. In other words, if each $A_i$ has cardinality strictly less than $kappa$, can we deduce that $bigcup_{i=1}^infty A_i$ has cardinality less than $kappa$?



I know very little cardinal arithmetic beyond the usual rules for sums and products. But I came across this in an analysis text, so I wonder whether it's true.










share|cite|improve this question









$endgroup$












  • $begingroup$
    What text was this #*!% in?
    $endgroup$
    – DanielWainfleet
    Jan 14 at 1:55


















2












$begingroup$


Is the following claim true?




If $bigcup_{i=1}^infty A_i$ has cardinality $kappa$, then some $A_i$ has cardinality $kappa$. Here $kappa$ is uncountable.




I'm interested in the particular case $kappa=c:=|mathbb{R}|$. In other words, if each $A_i$ has cardinality strictly less than $kappa$, can we deduce that $bigcup_{i=1}^infty A_i$ has cardinality less than $kappa$?



I know very little cardinal arithmetic beyond the usual rules for sums and products. But I came across this in an analysis text, so I wonder whether it's true.










share|cite|improve this question









$endgroup$












  • $begingroup$
    What text was this #*!% in?
    $endgroup$
    – DanielWainfleet
    Jan 14 at 1:55
















2












2








2





$begingroup$


Is the following claim true?




If $bigcup_{i=1}^infty A_i$ has cardinality $kappa$, then some $A_i$ has cardinality $kappa$. Here $kappa$ is uncountable.




I'm interested in the particular case $kappa=c:=|mathbb{R}|$. In other words, if each $A_i$ has cardinality strictly less than $kappa$, can we deduce that $bigcup_{i=1}^infty A_i$ has cardinality less than $kappa$?



I know very little cardinal arithmetic beyond the usual rules for sums and products. But I came across this in an analysis text, so I wonder whether it's true.










share|cite|improve this question









$endgroup$




Is the following claim true?




If $bigcup_{i=1}^infty A_i$ has cardinality $kappa$, then some $A_i$ has cardinality $kappa$. Here $kappa$ is uncountable.




I'm interested in the particular case $kappa=c:=|mathbb{R}|$. In other words, if each $A_i$ has cardinality strictly less than $kappa$, can we deduce that $bigcup_{i=1}^infty A_i$ has cardinality less than $kappa$?



I know very little cardinal arithmetic beyond the usual rules for sums and products. But I came across this in an analysis text, so I wonder whether it's true.







set-theory cardinals






share|cite|improve this question













share|cite|improve this question











share|cite|improve this question




share|cite|improve this question










asked Dec 31 '18 at 10:15









ColescuColescu

3,20011136




3,20011136












  • $begingroup$
    What text was this #*!% in?
    $endgroup$
    – DanielWainfleet
    Jan 14 at 1:55




















  • $begingroup$
    What text was this #*!% in?
    $endgroup$
    – DanielWainfleet
    Jan 14 at 1:55


















$begingroup$
What text was this #*!% in?
$endgroup$
– DanielWainfleet
Jan 14 at 1:55






$begingroup$
What text was this #*!% in?
$endgroup$
– DanielWainfleet
Jan 14 at 1:55












1 Answer
1






active

oldest

votes


















1












$begingroup$

Counter-example:



$$left| bigcup_{i=1}^infty omega_i right| = omega_omega$$





We have:



$$kappa = left| bigcup_{i=1}^infty A_i right| le omega cdot sup_{i=1..infty} |A_i| = sup_{i=1..infty} |A_i|$$



So in order to conclude that $kappa = |A_i|$ for some $i$, we need $operatorname{cof}(kappa) = omega$, otherwise there are coutner-examples. For $kappa = mathfrak c$ though, you are lucky, since $operatorname{cof}(mathfrak c) > omega$, so you can always conclude that $kappa = |A_i|$ for some $i$.





Specializing on the case $kappa = mathfrak c$:



Let $|A_i| < mathfrak c$ for all $i$. By Konig's lemma:
$$left| bigcup_{i=1}^infty A_i right| < left| prod_{i=1}^infty mathfrak c right| = mathfrak c^omega = 2^{omega cdot omega} = 2^omega = mathfrak c$$






share|cite|improve this answer











$endgroup$













  • $begingroup$
    Thanks for answering! Could you please elaborate on why this is true for $c$? I don't understand your notation cof and all that.. (I'm practically blind about set theory)
    $endgroup$
    – Colescu
    Dec 31 '18 at 10:25












  • $begingroup$
    The last part is not exactly true in ZF, it is consistent that the reals are countable union of countable sets. In ZFC it is true(the proof that $mbox{cof}(2^λ)>λ$ require choice)
    $endgroup$
    – Holo
    Dec 31 '18 at 10:33












  • $begingroup$
    @Colescu I've included some explanation
    $endgroup$
    – Kenny Lau
    Dec 31 '18 at 10:38










  • $begingroup$
    It might be better to wriite $aleph_i$ instead of $omega_i$, just to make sure we''re using cardinal operations. (For example: $omega=aleph_0$; one thinks of $omega$ as an ordinal annd $aleph_0$ as a cardinal. So, with cardinal multiplication, $aleph_0aleph_0=aleph_0$. But if the notation $omegaomega$ is interppreted as an ordinal product then $omegaomeganeomega$.)
    $endgroup$
    – David C. Ullrich
    Dec 31 '18 at 15:27













Your Answer





StackExchange.ifUsing("editor", function () {
return StackExchange.using("mathjaxEditing", function () {
StackExchange.MarkdownEditor.creationCallbacks.add(function (editor, postfix) {
StackExchange.mathjaxEditing.prepareWmdForMathJax(editor, postfix, [["$", "$"], ["\\(","\\)"]]);
});
});
}, "mathjax-editing");

StackExchange.ready(function() {
var channelOptions = {
tags: "".split(" "),
id: "69"
};
initTagRenderer("".split(" "), "".split(" "), channelOptions);

StackExchange.using("externalEditor", function() {
// Have to fire editor after snippets, if snippets enabled
if (StackExchange.settings.snippets.snippetsEnabled) {
StackExchange.using("snippets", function() {
createEditor();
});
}
else {
createEditor();
}
});

function createEditor() {
StackExchange.prepareEditor({
heartbeatType: 'answer',
autoActivateHeartbeat: false,
convertImagesToLinks: true,
noModals: true,
showLowRepImageUploadWarning: true,
reputationToPostImages: 10,
bindNavPrevention: true,
postfix: "",
imageUploader: {
brandingHtml: "Powered by u003ca class="icon-imgur-white" href="https://imgur.com/"u003eu003c/au003e",
contentPolicyHtml: "User contributions licensed under u003ca href="https://creativecommons.org/licenses/by-sa/3.0/"u003ecc by-sa 3.0 with attribution requiredu003c/au003e u003ca href="https://stackoverflow.com/legal/content-policy"u003e(content policy)u003c/au003e",
allowUrls: true
},
noCode: true, onDemand: true,
discardSelector: ".discard-answer"
,immediatelyShowMarkdownHelp:true
});


}
});














draft saved

draft discarded


















StackExchange.ready(
function () {
StackExchange.openid.initPostLogin('.new-post-login', 'https%3a%2f%2fmath.stackexchange.com%2fquestions%2f3057570%2fif-bigcup-i-1-infty-a-i-has-cardinality-kappa-then-some-a-i-has-card%23new-answer', 'question_page');
}
);

Post as a guest















Required, but never shown

























1 Answer
1






active

oldest

votes








1 Answer
1






active

oldest

votes









active

oldest

votes






active

oldest

votes









1












$begingroup$

Counter-example:



$$left| bigcup_{i=1}^infty omega_i right| = omega_omega$$





We have:



$$kappa = left| bigcup_{i=1}^infty A_i right| le omega cdot sup_{i=1..infty} |A_i| = sup_{i=1..infty} |A_i|$$



So in order to conclude that $kappa = |A_i|$ for some $i$, we need $operatorname{cof}(kappa) = omega$, otherwise there are coutner-examples. For $kappa = mathfrak c$ though, you are lucky, since $operatorname{cof}(mathfrak c) > omega$, so you can always conclude that $kappa = |A_i|$ for some $i$.





Specializing on the case $kappa = mathfrak c$:



Let $|A_i| < mathfrak c$ for all $i$. By Konig's lemma:
$$left| bigcup_{i=1}^infty A_i right| < left| prod_{i=1}^infty mathfrak c right| = mathfrak c^omega = 2^{omega cdot omega} = 2^omega = mathfrak c$$






share|cite|improve this answer











$endgroup$













  • $begingroup$
    Thanks for answering! Could you please elaborate on why this is true for $c$? I don't understand your notation cof and all that.. (I'm practically blind about set theory)
    $endgroup$
    – Colescu
    Dec 31 '18 at 10:25












  • $begingroup$
    The last part is not exactly true in ZF, it is consistent that the reals are countable union of countable sets. In ZFC it is true(the proof that $mbox{cof}(2^λ)>λ$ require choice)
    $endgroup$
    – Holo
    Dec 31 '18 at 10:33












  • $begingroup$
    @Colescu I've included some explanation
    $endgroup$
    – Kenny Lau
    Dec 31 '18 at 10:38










  • $begingroup$
    It might be better to wriite $aleph_i$ instead of $omega_i$, just to make sure we''re using cardinal operations. (For example: $omega=aleph_0$; one thinks of $omega$ as an ordinal annd $aleph_0$ as a cardinal. So, with cardinal multiplication, $aleph_0aleph_0=aleph_0$. But if the notation $omegaomega$ is interppreted as an ordinal product then $omegaomeganeomega$.)
    $endgroup$
    – David C. Ullrich
    Dec 31 '18 at 15:27


















1












$begingroup$

Counter-example:



$$left| bigcup_{i=1}^infty omega_i right| = omega_omega$$





We have:



$$kappa = left| bigcup_{i=1}^infty A_i right| le omega cdot sup_{i=1..infty} |A_i| = sup_{i=1..infty} |A_i|$$



So in order to conclude that $kappa = |A_i|$ for some $i$, we need $operatorname{cof}(kappa) = omega$, otherwise there are coutner-examples. For $kappa = mathfrak c$ though, you are lucky, since $operatorname{cof}(mathfrak c) > omega$, so you can always conclude that $kappa = |A_i|$ for some $i$.





Specializing on the case $kappa = mathfrak c$:



Let $|A_i| < mathfrak c$ for all $i$. By Konig's lemma:
$$left| bigcup_{i=1}^infty A_i right| < left| prod_{i=1}^infty mathfrak c right| = mathfrak c^omega = 2^{omega cdot omega} = 2^omega = mathfrak c$$






share|cite|improve this answer











$endgroup$













  • $begingroup$
    Thanks for answering! Could you please elaborate on why this is true for $c$? I don't understand your notation cof and all that.. (I'm practically blind about set theory)
    $endgroup$
    – Colescu
    Dec 31 '18 at 10:25












  • $begingroup$
    The last part is not exactly true in ZF, it is consistent that the reals are countable union of countable sets. In ZFC it is true(the proof that $mbox{cof}(2^λ)>λ$ require choice)
    $endgroup$
    – Holo
    Dec 31 '18 at 10:33












  • $begingroup$
    @Colescu I've included some explanation
    $endgroup$
    – Kenny Lau
    Dec 31 '18 at 10:38










  • $begingroup$
    It might be better to wriite $aleph_i$ instead of $omega_i$, just to make sure we''re using cardinal operations. (For example: $omega=aleph_0$; one thinks of $omega$ as an ordinal annd $aleph_0$ as a cardinal. So, with cardinal multiplication, $aleph_0aleph_0=aleph_0$. But if the notation $omegaomega$ is interppreted as an ordinal product then $omegaomeganeomega$.)
    $endgroup$
    – David C. Ullrich
    Dec 31 '18 at 15:27
















1












1








1





$begingroup$

Counter-example:



$$left| bigcup_{i=1}^infty omega_i right| = omega_omega$$





We have:



$$kappa = left| bigcup_{i=1}^infty A_i right| le omega cdot sup_{i=1..infty} |A_i| = sup_{i=1..infty} |A_i|$$



So in order to conclude that $kappa = |A_i|$ for some $i$, we need $operatorname{cof}(kappa) = omega$, otherwise there are coutner-examples. For $kappa = mathfrak c$ though, you are lucky, since $operatorname{cof}(mathfrak c) > omega$, so you can always conclude that $kappa = |A_i|$ for some $i$.





Specializing on the case $kappa = mathfrak c$:



Let $|A_i| < mathfrak c$ for all $i$. By Konig's lemma:
$$left| bigcup_{i=1}^infty A_i right| < left| prod_{i=1}^infty mathfrak c right| = mathfrak c^omega = 2^{omega cdot omega} = 2^omega = mathfrak c$$






share|cite|improve this answer











$endgroup$



Counter-example:



$$left| bigcup_{i=1}^infty omega_i right| = omega_omega$$





We have:



$$kappa = left| bigcup_{i=1}^infty A_i right| le omega cdot sup_{i=1..infty} |A_i| = sup_{i=1..infty} |A_i|$$



So in order to conclude that $kappa = |A_i|$ for some $i$, we need $operatorname{cof}(kappa) = omega$, otherwise there are coutner-examples. For $kappa = mathfrak c$ though, you are lucky, since $operatorname{cof}(mathfrak c) > omega$, so you can always conclude that $kappa = |A_i|$ for some $i$.





Specializing on the case $kappa = mathfrak c$:



Let $|A_i| < mathfrak c$ for all $i$. By Konig's lemma:
$$left| bigcup_{i=1}^infty A_i right| < left| prod_{i=1}^infty mathfrak c right| = mathfrak c^omega = 2^{omega cdot omega} = 2^omega = mathfrak c$$







share|cite|improve this answer














share|cite|improve this answer



share|cite|improve this answer








edited Dec 31 '18 at 10:36

























answered Dec 31 '18 at 10:19









Kenny LauKenny Lau

19.9k2160




19.9k2160












  • $begingroup$
    Thanks for answering! Could you please elaborate on why this is true for $c$? I don't understand your notation cof and all that.. (I'm practically blind about set theory)
    $endgroup$
    – Colescu
    Dec 31 '18 at 10:25












  • $begingroup$
    The last part is not exactly true in ZF, it is consistent that the reals are countable union of countable sets. In ZFC it is true(the proof that $mbox{cof}(2^λ)>λ$ require choice)
    $endgroup$
    – Holo
    Dec 31 '18 at 10:33












  • $begingroup$
    @Colescu I've included some explanation
    $endgroup$
    – Kenny Lau
    Dec 31 '18 at 10:38










  • $begingroup$
    It might be better to wriite $aleph_i$ instead of $omega_i$, just to make sure we''re using cardinal operations. (For example: $omega=aleph_0$; one thinks of $omega$ as an ordinal annd $aleph_0$ as a cardinal. So, with cardinal multiplication, $aleph_0aleph_0=aleph_0$. But if the notation $omegaomega$ is interppreted as an ordinal product then $omegaomeganeomega$.)
    $endgroup$
    – David C. Ullrich
    Dec 31 '18 at 15:27




















  • $begingroup$
    Thanks for answering! Could you please elaborate on why this is true for $c$? I don't understand your notation cof and all that.. (I'm practically blind about set theory)
    $endgroup$
    – Colescu
    Dec 31 '18 at 10:25












  • $begingroup$
    The last part is not exactly true in ZF, it is consistent that the reals are countable union of countable sets. In ZFC it is true(the proof that $mbox{cof}(2^λ)>λ$ require choice)
    $endgroup$
    – Holo
    Dec 31 '18 at 10:33












  • $begingroup$
    @Colescu I've included some explanation
    $endgroup$
    – Kenny Lau
    Dec 31 '18 at 10:38










  • $begingroup$
    It might be better to wriite $aleph_i$ instead of $omega_i$, just to make sure we''re using cardinal operations. (For example: $omega=aleph_0$; one thinks of $omega$ as an ordinal annd $aleph_0$ as a cardinal. So, with cardinal multiplication, $aleph_0aleph_0=aleph_0$. But if the notation $omegaomega$ is interppreted as an ordinal product then $omegaomeganeomega$.)
    $endgroup$
    – David C. Ullrich
    Dec 31 '18 at 15:27


















$begingroup$
Thanks for answering! Could you please elaborate on why this is true for $c$? I don't understand your notation cof and all that.. (I'm practically blind about set theory)
$endgroup$
– Colescu
Dec 31 '18 at 10:25






$begingroup$
Thanks for answering! Could you please elaborate on why this is true for $c$? I don't understand your notation cof and all that.. (I'm practically blind about set theory)
$endgroup$
– Colescu
Dec 31 '18 at 10:25














$begingroup$
The last part is not exactly true in ZF, it is consistent that the reals are countable union of countable sets. In ZFC it is true(the proof that $mbox{cof}(2^λ)>λ$ require choice)
$endgroup$
– Holo
Dec 31 '18 at 10:33






$begingroup$
The last part is not exactly true in ZF, it is consistent that the reals are countable union of countable sets. In ZFC it is true(the proof that $mbox{cof}(2^λ)>λ$ require choice)
$endgroup$
– Holo
Dec 31 '18 at 10:33














$begingroup$
@Colescu I've included some explanation
$endgroup$
– Kenny Lau
Dec 31 '18 at 10:38




$begingroup$
@Colescu I've included some explanation
$endgroup$
– Kenny Lau
Dec 31 '18 at 10:38












$begingroup$
It might be better to wriite $aleph_i$ instead of $omega_i$, just to make sure we''re using cardinal operations. (For example: $omega=aleph_0$; one thinks of $omega$ as an ordinal annd $aleph_0$ as a cardinal. So, with cardinal multiplication, $aleph_0aleph_0=aleph_0$. But if the notation $omegaomega$ is interppreted as an ordinal product then $omegaomeganeomega$.)
$endgroup$
– David C. Ullrich
Dec 31 '18 at 15:27






$begingroup$
It might be better to wriite $aleph_i$ instead of $omega_i$, just to make sure we''re using cardinal operations. (For example: $omega=aleph_0$; one thinks of $omega$ as an ordinal annd $aleph_0$ as a cardinal. So, with cardinal multiplication, $aleph_0aleph_0=aleph_0$. But if the notation $omegaomega$ is interppreted as an ordinal product then $omegaomeganeomega$.)
$endgroup$
– David C. Ullrich
Dec 31 '18 at 15:27




















draft saved

draft discarded




















































Thanks for contributing an answer to Mathematics Stack Exchange!


  • Please be sure to answer the question. Provide details and share your research!

But avoid



  • Asking for help, clarification, or responding to other answers.

  • Making statements based on opinion; back them up with references or personal experience.


Use MathJax to format equations. MathJax reference.


To learn more, see our tips on writing great answers.




draft saved


draft discarded














StackExchange.ready(
function () {
StackExchange.openid.initPostLogin('.new-post-login', 'https%3a%2f%2fmath.stackexchange.com%2fquestions%2f3057570%2fif-bigcup-i-1-infty-a-i-has-cardinality-kappa-then-some-a-i-has-card%23new-answer', 'question_page');
}
);

Post as a guest















Required, but never shown





















































Required, but never shown














Required, but never shown












Required, but never shown







Required, but never shown

































Required, but never shown














Required, but never shown












Required, but never shown







Required, but never shown







Popular posts from this blog

Cabo Verde

Gyllenstierna

Karlovacs län